Kysymys:
Planeettojen sähköinen potentiaaliero
Kavka
2020-02-07 07:49:02 UTC
view on stackexchange narkive permalink

Käsitellessämme sähköä tarkoitamme maapalloa yleensä sähköneutraalina tai maana. Onko mitään todisteita siitä, että muiden planeettojen maa on samalla sähköpotentiaalitasolla kuin maapallon? Olisiko tämä teoreettisesti energialähde?

Jos liität sähköisesti neutraalin ja sähköisesti neutraalin, saat ...
Erilaiset mutta liittyvät: [Mitkä ovat kokeelliset rajat auringon jäännösvaraukseen?] (Https://astronomy.stackexchange.com/q/23319/7982) Tämä on mielenkiintoinen kysymys; planeetan nettovaraus ei välttämättä ole nolla; koska elektronien ja ionien liikkuvuus on niin erilaista, mukana on magneettikenttiä ja jatkuva ionisoitujen hiukkasten syöttö auringosta. Voisit harkita kysymystä erityisesti maapallon nettovarauksesta erillisenä kysymyksenä julkaisussa [Earth Science SE] (https://earthscience.stackexchange.com/).
Olen löytänyt myös hieman liittyvän kysymyksen täältä: https://physics.stackexchange.com/questions/253475/do-stars-remain-electrically-neutral.
Kavka, älä unohda mahdollisuutta lähettää oma vastauksesi tähän kysymykseen, jos huomaat, että muiden kommenttien linkit vievät sinut ratkaisuun. Stack Exchangessa kannustetaan itse vastaamaan - itse asiassa siellä on [erityinen merkki, jonka voit ansaita] (https://astronomy.stackexchange.com/help/badges/14/self-learner)! :-)
üks vastaus:
Carl Witthoft
2020-02-07 22:54:04 UTC
view on stackexchange narkive permalink

En tiedä miten tai miksi planeetalla olisi merkittävä nettovaraus, mutta jos oletetaan, että tässä on, tässä on joitain alkuarvioita.

Jos planeetoilla on erilainen nettovaraus, voit kohdella niitä kondensaattorilevyinä.
Kapasitanssi on yksinkertaisimmillaan

$ C = \ frac {\ epsilon * A} {d} $
Tyhjiötilan läpäisykyky $ \ epsilon $ on 8,85 x 10- 12 faradia metriä kohti. Valitaan lähellä oleva (olematon) planeetta 5 LY = 5 * 9,461e + 15 m.

Näyttää siltä, ​​että kapasitanssi on melko lähellä nollaa. Joten käytettävissä olevaa energiaa,

$ E = \ frac {CV ^ 2} {2} $ , ei tule olemaan, ellei siellä ole vakavasti suurta määrää Jännite.



Tämä Q & A käännettiin automaattisesti englanniksi.Alkuperäinen sisältö on saatavilla stackexchange-palvelussa, jota kiitämme cc by-sa 4.0-lisenssistä, jolla sitä jaetaan.
Loading...